Вы находитесь на странице: 1из 6

Solutions to Chapter 5 exercises

5.1 From the triangle inequality d(x, z)  d(x, y) + d(y, z), so d(x, z) − d(y, z)  d(x, y).
From the triangle inequality and symmetry d(y, z)  d(y, x) + d(x, z) = d(x, y) + d(x, z), so
d(y, z) − d(x, z)  d(x, y). Together these give |d(x, z) − d(y, z)|  d(x, y).

5.2 Using the triangle inequality for R and symmetry,

|d(x, y) − d(z, t)| = |d(x, y) − d(y, z) + d(y, z) − d(z, t)|  |d(x, y) − d(y, z)| + |d(y, z) − d(z, t)|

= |d(x, y) − d(z, y)| + |d(y, z) − d(t, z)|  d(x, z) + d(y, t)

where the last inequality uses Exercise 5.1.

5.3 The proof is by induction on n. For n = 3 it is the triangle inequality. Suppose that it
is true for a given integer value of n  3. Then d(x1 , xn+1 )  d(x1 , xn ) + d(xn , xn+1 ) by the
triangle inequality. From this, using the inductive hypothesis, we get

d(x1 , xn+1 )  d(x1 , x2 ) + d)x2 , x3 ) + . . . + d(xn−1 , xn ) + d(xn , xn+1 ),

which tells us that the formula is true for n + 1. By induction it is true for all integers n  3.

5.4 It is clear for all three cases (a), (b), (c) that d(x, x) = 0 and that symmetry holds (in view
of the mod signs). To see that d(x, y) = 0 implies x = y , we note that each of the functions
x → x3 , x → ex , x → tan−1 (x) is strictly monotonic increasing, and this gives what we want.
The triangle inequality follows for each d from the triangle inequality for R: for example in (a),

d(x, z) = |x3 − z 3 | = |x3 − y 3 + y 3 − z 3 |  |x3 − y 3| + |y 3 − z 3 | = d(x, y) + d(y, z).

What makes this work is the strictly monotonic increasing property of the functions: if
f : R → R is any strictly monotonic increasing function then the formula d(x, y) = |f (x) − f (y)|
defines a metric for R.

5.5 Suppose for a contradiction that z ∈ Bε (x) ∩ Bε (y). Then d(z, x) < ε and d(z, y) < ε, so
by the triangle inequality and symmetry, d(x, y)  d(x, z) + d(z, y) = d(z, x) + d(z, y) < 2ε,
contradicting the fact that d(x, y) = 2ε.

5.6 With the hypotheses of the question, suppose that z ∈ Bε/2 (y). Then

d(z, x)  d(z, y) + d(y, x) < ε/2 + ε/2 = ε

so z ∈ Bε (x). This shows that Bε/2 (y) ⊆ Bε (x).


5.7 Since S is bounded, for some (a1 , a2 , . . . , an ) ∈ Rn and some K ∈ R we have

(x1 − a1 )2 + (x2 − a2 )2 + . . . + (xn − an )2  K for all (x1 , x2 , . . . , xn ) ∈ S.

In particular, for each i = 1, 2, . . . , |xi − ai |  K so xi ∈ [ai − K, ai + K]. Let a = m − K


and b = K + M where m = min{ai : i = 1, 2, . . . n}, M = max{ai : i = 1, 2, . . . , n}. Then
xi ∈ [a, b] for each i = 1, 2, 3 . . . n, so (x1 , x2 , . . . , xn ) ∈ [a, b] × [a, b] × . . . × [a, b] (product
of n copies of [a, b]). This holds for all (x1 , x2 , . . . xn ) in S , so S ⊆ [a, b] × [a, b] × . . . × [a, b]
(product of n copies of [a, b]).

5.8 Recall that the definition of a bounded set A in a metric space X is that there exists some
x0 ∈ X and some real number K such that d(a, x0 )  K for all a ∈ A. If there exists ∆ ∈ R
such that d(a, a )  ∆ for all a, a ∈ A, then A is bounded since we can take x0 to be any
point in A and K to be ∆.
Conversely suppose that A is bounded, so there exists x0 ∈ X and K ∈ R such that
d(a, x0 )  K for all a ∈ A. Put ∆ = 2K . Then for any a, a ∈ A we have

d(a, a )  d(a, x0 ) + d(x0 , a )  2K = ∆ as required.

5.9 Let the metric space be X . There exists some x0 ∈ X and some K ∈ R such that
d(b, x0 )  K for all b ∈ B . Since A ⊆ B this holds in particular for all points in A, so A is
bounded.
If A = ∅ then by definition diamA = 0 so certainly diamA  diamB , since the latter is
the sup of a set of non-negative numbers. Now suppose A = ∅. For all b, b ∈ B we have
d(b, b ) diamB , so in particular d(a, a )  diamB for any a, a ∈ A. Since diamA is the sup
of such distances, diamA  diamB .

5.10 Let y ∈ A ∩ B . For any x, x ∈ A ∪ B we have


d(x, x )  d(x, y) + d(y, x )  diam A + diam B,

so diam(A ∪ B)  diamA+ diamB , by definition of diam(A ∪ B).


6

r -
(1, 0)

Fig. 1

5.11 Since d∞ ((x, y), (0, 0)) = max{|x|, |y|}, we see that (x, y) ∈ B1d∞ ((0, 0)) iff −1 < x < 1
and also −1 < y < 1, so the unit ball is as shown in Fig. 1.

5.12 The metric space axioms all hold for d(1) since they hold for d.
It is clear that d(2) (x, x) = d(3) (x, x) = 0. Suppose d(2) (x, y) = 0. Then we must have
d(2) (x, y) = d(x, y), since 0 < 1. This shows that d(x, y) = 0 so x = y. If d(3) (x, y) = 0 then
d(x, y) = 0 so x = y . It is clear from the formulas that d(2) and d(3) satisfy the symmetry
axiom.
We now check the triangle inequality for d(2) . For points x, y, z we have

d(2) (x, z) = min{1, d(x, z)}  min{1, d(x, y) + d(y, z)}

 min{1, d(x, y)} + min{1, d(y, z)} = d(2) (x, y) + d(2) (y, z).

Finally, to check the triangle inequality for d(3) we want to show that if a, b, c are positive real
numbers with a + b  c then
a b c
+  .
1+a 1+b 1+c
(Then we get the result by taking a = d(x, y), b = d(y, z), c = d(x, z).) Now

a b a + b + 2ab d
+ =  where d = a + b + 2ab.
1+a 1+b (1 + a)(1 + b) 1+d

Now d  a + b  c, so it is enough to show that the function f defined by f (x) = x/(1 + x) is


increasing for x > 0. But f  (x) = 1/(1 + x)2 which is positive for x > 0, and we have completed
the proof of the triangle inequality for d(3) .
If d is the discrete metric on any set then d(4) (x, y) = d(x, y)2 = d(x, y) so in this case d(4)
is a metric. But if d is the usual metric on R then d(4) is not a metric: if for example we take
x = 0, y = 1/2, z = 1 then d(4) (x, y) = 1/4, d(4) (y, z) = 1/4, but d(4) (x, z) = 1, and in this
case d(4) (x, y) + d(4) (y, z) < d(4) (x, z), so the triangle inequality fails for d(4) .

5.13 Since any open ball is an open set by Proposition 5.31, any union of open balls is an open
set by Proposition 5.41.
Conversely, given an open set in a metric space X , for each x ∈ U by definition there exists
εx > 0 such that Bεx (x) ⊆ U. Then

U= Bεx (x).
x∈U
For since each Bεx (x) ⊆ U , their union is contained in U . Also, any x ∈ U is in Bεx (x) and
hence is in the union on the right-hand side.

5.14 For any x = (x1 , x2 , . . . , xn ), y = (y1 , y2 , . . . , yn ) ∈ Rn we have



 n
 
d∞ (x, y) = max |xi − yi |   (xi − yi )2  |xi − y1 |  n max |xi − yi | = nd∞ (x, y),
1in 1in
i=1 1in
which says
d∞(x, y)  d2 (x, y)  d1 (x, y)  nd∞ (x, y) as required.


5.15(a) If y ∈ Bε/k
d
(x) then d (y, x) < ε/k , so d(x, y)  kd (x, y) < ε, so y ∈ Bεd (x). This

d
shows that Bε/k (x) ⊆ Bεd (x).
(b) If U is d-open, then for any x ∈ U there exists ε > 0 such that Bεd (x) ⊆ U. Then

d
Bε/k (x) ⊆ Bεd (x) ⊆ U. So U is d -open.
(c) This follows from (b) together with Exercise 5.14.

5.16(a) Since dX , dY are metrics, it is clear that di ((x1 , y1 ), (x1 , y1 )) = 0 for i = 1, 2, ∞.


If d1 ((x1 , y1 ), (x2 , y2 )) = 0 then dX (x1 , x2 ) = 0 = dY (y1 , y2 ), so x2 = x1 and y2 = y1 again
since dX , dY are metrics. This says (x2 , y2 ) = (x1 y1 ).
The same argument shows that if di ((x1 , y1 ), (x2 , y2 )) = 0 with i = 2 or i = ∞ then
(x2 , y2 ) = (x1 , y1 ).
Symmetry of di for i = 1, 2, ∞ is clear since dX and dY are symmetric.
If (x1 , y1 ), (x2 , y2 ), (x3 , y3 ) ∈ X × Y then

d1 ((x1 , y1 ), (x3 , y3 )) = dX (x1 , x3 )+dY (y1 , y3 )  dX (x1 , x2 )+dX (x2 , x3 )+dY (y1, y2 )+dY (y2 , y3 )

= d1 ((x1 , y1 ), (x2 , y2 )) + d1 ((x2 , y2 ), (x3 , y3 )),

which checks the triangle inequality for d1 .


We now check the triangle inequality for d∞ . For (x1 , y1 ), (x2 , y2 ), (x3 , y3 ) ∈ X × Y we
have

dX (x1 , x3 )  dX (x1 , x2 ) + dX (x2 , x3 )  d∞ ((x1 , y1 ), (x2 , y2 )) + d∞ ((x2 , y2 ), (x3 , y3 )).

Similarly dY (y1 , y3 )  d∞ ((x1 , y1 ), (x2 , y2 )) + d∞ ((x2 , y2 ), (x3 , y3 )), so


d∞ ((x1 , y1 ), (x3 , y3 )) = max{dX (x1 , x3 ), dY (y1 , y3 )}

 d∞ ((x1 , y1 ), (x2 , y2 )) + d∞ ((x2 , y2 ), (x3 , y3 )).

Finally we check the triangle inequality for d2 . For (x1 , y1 ), (x2 , y2 ), (x3 , y3 ) ∈ X × Y we
have

d2 ((x1 , y1 ), (x3 , y3 )) = dX (x1 , x3 )2 + dY (y1 , y3 )2

 (dX (x1 , x2 ) + dX (x2 , x3 ))2 + (dY (y1 , y2 ) + dY (y2 , y3 ))2 .
 
We want this not to exceed dX (x1 , x2 )2 + dY (y1 , y2 )2 + dX (x2 , x3 )2 + dY (y2 , y3 )2 . This is
a matter of proving
 √ √
(a + b)2 + (c + d)2  a2 + c2 + b2 + d2 for non-negative real numbers a, b, c, d.

It is equivalent to prove

(a + b)2 + (c + d)2  a2 + c2 + b2 + d2 + 2 (a2 + c2 )(b2 + d2 ),

or equivalently (ab + cd)  (a2 + c2 )(b2 + d2 ), or equivalently (ab + cd)2  (a2 + c2 )(b2 + d2 ),
or equivalently 2abcd  a2 d2 + b2 c2 . But this is true since (ad − bc)2  0.
5.16(b) Let p = (x1 , y1 ), q = (x2 , y2 ). Then

max{dX (x1 , x2 ), dY (y1 , y2 )}  dX (x1 , x2 )2 + dY (y1 , y2 )

 dX (x1 , x2 ) + dY (y1 , y2 )  2 max{dX (x1 , x2 ), dY (y1 , y2 )},

where the second inequality follows from since it holds for the squares of the given quantities.
The above gives the required result.
5.16(c) This is exactly like Exercise 5.15(b).
5.16(d) Let U, V be open in X, Y respectively. Let (x0 , y0 ) ∈ U × V . Since U is dX -open
in X , there exists ε1 > 0 such that Bεd1X (x0 ) ⊆ U . Similarly there exists ε2 > 0 such that
Bεd2Y (y0 ) ⊆ V . Let ε = min{ε1 , ε2 }.
To see that U × V is d1 -open, we prove Bεd1 ((x0 , y0 )) ⊆ U × V . For d1 ((x, y), (x0 , y0 )) < ε
says dX (x, x0 ) + dY (y, y0 ) < ε, and then dX (x, x0 )  d1 ((x, y), (x0 , y0 )) < ε1 so x ∈ U , and
similarly y ∈ V .
Next we prove that U × V is d2 -open. For (x0 , y0 ) ∈ U × V let ε be as above. We have

d2 ((x, y), (x0 , y0 )) = dX (x, x0 )2 + dY (y, y0 )2 . Hence whenever d2 ((x, y), (x0 , y0 )) < ε, we
have dX (x, x0 ) < ε  ε1 so x ∈ U and similarly y ∈ V . So Bεd2 ((x0 , y0 )) ⊆ U × V, and U × V
is d2 -open.
Finally we prove that U × V is d∞ -open. Again for (x0 , y0 ) ∈ U × V let ε be as above. We
have d∞ ((x, y), (x0 , y0 )) = max{dX (x, x0 ), dY (y, y0 )}. So if d∞ ((x, y), (x0 , y0 )) < ε then

dX (x, x0 )  max{dX (x, x0 ), dY (y, y0 )} = d∞ ((x, y), (x0 , y0 )) < ε  ε1

so x ∈ U ; similarly y ∈ V . So Bεd∞ ((x0 , y0 )) ⊆ U × V, and U × V is d∞ -open.

5.17 Let x, y, z, t ∈ X . From Exercise 5.2, |d(x, y)−d(z, t)|  d(x, z)+d(y, t) = d1 ((x, y), (z, t)).
So given ε > 0 we may take δ = ε, and if d1 ((x, y), (z, t)) < δ then |d(x, y) − d(z, t)| < δ = ε,
so d : X × X → R is continuous.

5.18 We consider a discrete space X containing at least two distinct points x, y . Take any
r > s > 1. Then Br (x) = X = Bs (y) yet x = y and also r = s. Hence in general we need not
have x = y or r = s when Br (x) = Bs (y).

Вам также может понравиться